Đến nội dung

Hình ảnh

Topic về bất đẳng thức

* * * * * 16 Bình chọn

  • Please log in to reply
Chủ đề này có 206 trả lời

#161
khanhhx

khanhhx

    Binh nhất

  • Thành viên
  • 24 Bài viết

77. Cho $a,b,c\geq 0$$: a^2+b^2+c^2+abc=4.$ CMR: $\frac{a}{b^{2}+2c}+\frac{b}{c^{2}+2a}+\frac{c}{a^{2}+2b}\geqslant 1$


Bài viết đã được chỉnh sửa nội dung bởi E. Galois: 14-07-2015 - 08:16


#162
duongvanhehe

duongvanhehe

    Trung sĩ

  • Thành viên
  • 117 Bài viết

Cho $a,b,c\geq 0$$: a^2+b^2+c^2+abc=4.$ CMR: $\frac{a}{b^{2}+2c}+\frac{b}{c^{2}+2a}+\frac{c}{a^{2}+2b}\geqslant 1$

Áp dụng BĐT Cauchy-Schwarz:
$VT\geq \frac{(a+b+c)^{2}}{ab^{2}+bc^{2}+ca^{2}+2(ab+bc+ca)}$
Giờ ta chỉ cần chứng minh được:$a^{2}+b^{2}+c^{2}\geq ab^{2}+bc^{2}+ca^{2}$
Ta có $(ab^{2}+bc^{2}+ca^{2})^{2}\leq (ab+bc+ca)(ab^{3}+bc^{3}+ca^{3})$
Ta có kết quả quen thuộc $3(ab^{3}+bc^{3}+ca^{3})\leq (a^{2}+b^{2}+c^{2})^{2}$
nên ta phải chứng minh được $ab+bc+ca\leq 3$
$\cdot nếu a+b+c\leq 3$ ta có ngay $ab+bc+ca\leq 3$
$\cdot nếu a+b+c\geq 3$ từ điều kiện ta suy ra được $abc\leq 1$ nên $6\geq a^{2}+b^{2}+c^{2}+2abc+1\geq a^{2}+b^{2}+c^{2}+\frac{9abc}{a+b+c}\geq 2(ab+bc+ca)$(BĐT Schur )
Suy ra đpcm :icon6:
FC.Fruit

#163
ducthinh26032011

ducthinh26032011

    Thượng sĩ

  • Thành viên
  • 290 Bài viết

Bài toán 78:Cho $a,b,c\in R^{+}$ thỏa $a^{2}+b^{2}+c^{2}=3$.C/m:
$$\frac{a}{b}+\frac{b}{c}+\frac{c}{a}\geq \frac{9}{a+b+c}$$


Bài viết đã được chỉnh sửa nội dung bởi E. Galois: 13-08-2015 - 15:37

Hình đã gửi


#164
WhjteShadow

WhjteShadow

    Thượng úy

  • Phó Quản lý Toán Ứng dụ
  • 1323 Bài viết

Bài toán:Cho $a,b,c\in R^{+}$ thỏa $a^{2}+b^{2}+c^{2}=3$.C/m:
$$\frac{a}{b}+\frac{b}{c}+\frac{c}{a}\geq \frac{9}{a+b+c}$$

Áp dụng bất đẳng thức $Cauchy-Schwarz$ ta có:
$$\frac{a}{b}+\frac{b}{c}+\frac{c}{a}=\frac{a^2}{ab}+\frac{b^2}{bc}+\frac{c^2}{ca}\geq \frac{(a+b+c)^2}{ab+bc+ca}$$
Vậy ta chỉ cần chứng minh: $\frac{(a+b+c)^2}{ab+bc+ca}\geq \frac{9}{a+b+c}$
$$\Leftrightarrow (a+b+c)^3\geq 9(ab+bc+ca)$$
$$\Leftrightarrow 2(a+b+c)^3\geq 9[2(ab+bc+ca)]$$
$$\Leftrightarrow 2(a+b+c)^3\geq 9[(a+b+c)^2-3]$$
$$\Leftrightarrow (a+b+c)^3+(a+b+c)^3+27 \geq 9(a+b+c)^2$$
Nhưng bất đẳng thức cuối lại luôn đúng the0 $AM-GM$ 3 số nên ta có ĐPCM.
ĐẲng thức xảy ra $\Leftrightarrow a=b=c=1$

Bài viết đã được chỉnh sửa nội dung bởi WhjteShadow: 04-09-2012 - 21:36

“There is no way home, home is the way.” - Thich Nhat Hanh

#165
viet 1846

viet 1846

    Gà con

  • Thành viên
  • 224 Bài viết

Bài toán:Cho $a,b,c\in R^{+}$ thỏa $a^{2}+b^{2}+c^{2}=3$.C/m:
$$\frac{a}{b}+\frac{b}{c}+\frac{c}{a}\geq \frac{9}{a+b+c}$$


Nguồn Boxmath.vn:


Giả sử $c = m{\rm{ax}}\left\{ {a;b;c} \right\}$
$ \Rightarrow a \le 1$
Nếu $a \ge \frac{3}{2}$ thì ${a^2} + {c^2} \ge 3,25 \Rightarrow a < \frac{3}{2}$

\[3 = {a^2} + {b^2} + {c^2} \ge 3\sqrt {{a^2}{b^2}{c^2}} \Rightarrow abc \le 1\]

\[ineq \Leftrightarrow M{\left( {a - b} \right)^2} + N\left( {a - c} \right)\left( {b - c} \right) \ge 0\]

Với \[M = \frac{1}{{ab}} - \frac{6}{{\left( {a + b + c} \right)\left( {a + b + c + 3} \right)}}\]

\[N = \frac{1}{{ac}} - \frac{6}{{\left( {a + b + c} \right)\left( {a + b + c + 3} \right)}}\]

Ta có: \[M \ge 1 - \frac{6}{{\sqrt 3 \left( {3 + \sqrt 3 } \right)}} > 0\]

\[N = \frac{{{{\left( {a - c} \right)}^2} + c\left( {3 - 2a} \right) + b\left( {b + 2a + 2c} \right)}}{{ac\left( {a + b + c} \right)\left( {a + b + c + 3} \right)}} > 0\]

BĐT được chứng minh. :D



Mình có thêm 1 cách nữa:
$BDT\iff (a+b+c)(\frac{a}{b}+\frac{b}{c}+\frac{c}{a})\ge 9$
$\iff (\frac{a^2}{b}+\frac{b^2}{c}+\frac{c^2}{a})+(\frac{ac}{b}+\frac{ba}{c}+\frac{cb}{a})+(a+b+c)\ge 9$
Ta có các BĐT
$*\frac{a^2}{b}+\frac{b^2}{c}+\frac{c^2}{a}\ge \frac{(a^2+b^2+c^2)(a+b+c)}{ab+bc+ca}$
[spoiler=CM:]Chứng minh bằng cách nhân vế theo vế với $ab+bc+ca$ rồi thu gọn và sử dụng $AM-GM$ từng cặp 1.[/spoiler]
$*\frac{ac}{b}+\frac{ba}{c}+\frac{cb}{a}\ge \sqrt{3(a^2+b^2+c^2)}=3$
Do đó
$VT\ge \frac{3(a+b+c)}{ab+bc+ca}+(a+b+c)+3\ge 2\sqrt{\frac{3(a+b+c)^2}{ab+bc+ca}}+3\ge 2.3+3=9$
DPCM!



Cho điều kiện mạnh hơn, bdt còn đúng với $a^2+b^2+c^2 \ge 3 $

Xét các số thực dương $x;y;z$ sao cho $x^2+y^2+z^2=3 $

Tồn tại số thực dương $t \ge 1$ sao cho $a=tx; b=ty;c =tz $

Ta cần cm

$ \frac{x}{y}+\frac{y}{z}+\frac{z}{x} \ge \frac{9}{t(x+y+z)} $

Ta đã cm được $\frac{x}{y}+\frac{y}{z}+\frac{z}{x} \ge \frac{9}{x+y+z} \ge \frac{9}{t(x+y+z)} $



Các bạn thử làm bài mạnh hơn này xem nhé:
Chứng minh với cùng điều kiện:
$$\frac{a}{b}+\frac{b}{c}+\frac{c}{a}\geq \frac{27}{(a+b+c)^2}$$


Lâu lắm mới động đến BĐT.

#166
thuan192

thuan192

    Sĩ quan

  • Thành viên
  • 325 Bài viết

Tiếp tục nào image001.gif
Bài 9: Cho các số thực không âm $a,b,c$ thỏa mãn$a+b+c=2$. CMR:

$\dfrac{{bc}}{{{a^2} + 1}} + \dfrac{{ac}}{{{b^2} + 1}} + \dfrac{{ab}}{{{c^2} + 1}} \leqslant 1$


*Nhận xét : Bài này có khá nhiều cách giải rất trâu bò
Các bạn có thể tìm đc bao nhiêu cách giải cho bài này ??? image001.gif



Các bạn nhanh chóng giải bài này đi image001.gif
Gợi ý : Chỉ sử dụng bất đẳng thức $AM-GM$ $(Cauchy)$

Đừng post bài nữa nhá image001.gif để giải xong mấy bài trên rồi post tiếp nha image004.gif

 

bạn xem ở đây   http://diendantoanho...b-1-le-8/page-2


:lol:Thuận :lol:

#167
thuan192

thuan192

    Sĩ quan

  • Thành viên
  • 325 Bài viết

$cho 3 số thực a, b, c thỏa a^{3}> 36;abc=1. CMR \frac{a^{2}}{3}+b^{2}+c^{2}> ab+bc+ca$

Ta có hiệu : $\frac{a^{2}}{3}+b^{2}+c^{2}-ab-bc-ac$

                    $= \frac{a^{2}}{4}+\frac{a^{2}}{12}+b^{2}+c^{2}-ab-bc-ca$

                    $= \left ( \frac{a^{2}}{4}+b^{2}+c^{2}-ab-ac+2bc \right )+\frac{a^{2}}{12}-3bc$

                    $= \left ( \frac{a}{2}-b-c \right )^{2}+\frac{a^{3}-36abc}{12a}$

                    $= \left ( \frac{a}{2}-b-c \right )^{2}+\frac{a^{3}-36}{12a}> 0$

 Suy ra đpcm


:lol:Thuận :lol:

#168
thuan192

thuan192

    Sĩ quan

  • Thành viên
  • 325 Bài viết

$cho 3 số thực a, b, c thỏa a^{3}> 36;abc=1. CMR \frac{a^{2}}{3}+b^{2}+c^{2}> ab+bc+ca$

Tiếp một cách nữa:  

   Từ giả thiết $a^{3}> 36$ suy ra a>0 và $bc=\frac{1}{a}$

    Bất đẳng thức cần chứng minh viết dưới dạng 

               $\frac{a^{2}}{3}+b^{2}+c^{2}+2bc-2bc> a\left ( b+c \right )+bc$

               $\Leftrightarrow \frac{a^{2}}{3}+\left ( b+c \right )^{2}-3bc-a\left ( b+c \right )>0$

               $\Leftrightarrow \left ( b+c \right )^{2}-a\left ( b+c \right )+\frac{a^{2}}{3}-\frac{3}{a}>0 (*)$

   Dặt t=b+c, Xét $f\left ( t \right )=t^{2}-at+\frac{a^{2}}{3}-\frac{3}{a}$   (với a^3> 36)

   ta lập $\Delta$ và chứng minh  $\Delta < 0$

                  và suy ra đpcm


:lol:Thuận :lol:

#169
Silverbullet069

Silverbullet069

    Thiếu úy

  • Thành viên
  • 565 Bài viết

cho a,b,c>0, a+b+c=3.CMR

$\sqrt{a}+\sqrt{b}+\sqrt{c} \geq  ab+bc+ca$

Áp dụng BĐT AM-GM, ta có :

$a^2 + \sqrt{a} + \sqrt{a} \geq 3a$

$b^2 + \sqrt{b} + \sqrt{b} \geq 3b$

$c^2 + \sqrt{c} + \sqrt{c} \geq 3c$

Cộng vế với vế :

$\Rightarrow a^2 + b^2 + c^2 + 2(\sqrt{a} + \sqrt{b} + \sqrt{c}) \geq 3(a + b + c) = 3.3 = 9$

$\Rightarrow 2(\sqrt{a} + \sqrt{b} + \sqrt{c}) \geq 9 - (a^2 + b^2 + c^2)$

$\Rightarrow 2(\sqrt{a} + \sqrt{b} + \sqrt{c}) \geq 9 - [(a + b + c)^2 - 2(ab + bc + ca)] = 9 - (a + b + c)^2 + 2(ab + bc + ca)] = 9 - 9 + 2(ab + bc + ca) = 2(ab + bc + ca)$

$\Rightarrow \sqrt{a} + \sqrt{b} + \sqrt{c} \geq ab + bc + ca$

Dấu "=" xảy ra $\Leftrightarrow a = b = c = 1$

Nguồn : vn.answers.yahoo.com


Bài viết đã được chỉnh sửa nội dung bởi Silverbullet069: 13-08-2015 - 15:49

"I am the bone of my sword,

 

Unknown to Death, Nor known to Life,

 

So as I pray, unlimited blade works."

 

 


#170
Longtunhientoan2k

Longtunhientoan2k

    Thượng sĩ

  • Thành viên
  • 272 Bài viết

Áp dụng BĐT AM-GM, ta có :

$a^2 + \sqrt{a} + \sqrt{a} \geq 3a$

$b^2 + \sqrt{b} + \sqrt{b} \geq 3b$

$c^2 + \sqrt{c} + \sqrt{c} \geq 3c$

Cộng vế với vế :

$\Rightarrow a^2 + b^2 + c^2 + 2(\sqrt{a} + \sqrt{b} + \sqrt{c}) \geq 3(a + b + c) = 3.3 = 9$

$\Rightarrow 2(\sqrt{a} + \sqrt{b} + \sqrt{c}) \geq 9 - (a^2 + b^2 + c^2)$

$\Rightarrow 2(\sqrt{a} + \sqrt{b} + \sqrt{c}) \geq 9 - [(a + b + c)^2 - 2(ab + bc + ca)] = 9 - (a + b + c)^2 + 2(ab + bc + ca)] = 9 - 9 + 2(ab + bc + ca) = 2(ab + bc + ca)$

$\Rightarrow \sqrt{a} + \sqrt{b} + \sqrt{c} \geq ab + bc + ca$

Dấu "=" xảy ra $\Leftrightarrow a = b = c = 1$

Nguồn : vn.answers.yahoo.com

Bài này em thấy giống một bài nào đó trong sáng tạo bất đẳng thức của anh Phạm Kim Hùng


         LONG VMF NQ MSP 


#171
duythanbg

duythanbg

    Hạ sĩ

  • Thành viên
  • 77 Bài viết

Bài toán :  Cho a,b,c là các số thực dương . CMR : 

                   $\frac{a}{b}+\frac{b}{c}+\frac{c}{a}\geq \frac{9(a^2+b^2+c^2)}{(a+b+c)^2}$

 

P/s : Mình đã nghĩ rất lâu mà không giải được. :angry:


          

 

 

 


#172
Longtunhientoan2k

Longtunhientoan2k

    Thượng sĩ

  • Thành viên
  • 272 Bài viết

 Bạn thử sử dụng kĩ thuật chauanr hóa chưa


         LONG VMF NQ MSP 


#173
duythanbg

duythanbg

    Hạ sĩ

  • Thành viên
  • 77 Bài viết

Rồi.Mình dùng hết các pp rồi ( chắc đánh giá kém ). Mình chẳng tìm thấy lời giải ở đâu cả.  :(


          

 

 

 


#174
hoctrocuaHolmes

hoctrocuaHolmes

    Thượng úy

  • Thành viên
  • 1013 Bài viết

Bài toán :  Cho a,b,c là các số thực dương . CMR : 
                   $\frac{a}{b}+\frac{b}{c}+\frac{c}{a}\geq \frac{9(a^2+b^2+c^2)}{(a+b+c)^2}$
 
P/s : Mình đã nghĩ rất lâu mà không giải được. :angry:

 Chuẩn hoá $a+b+c=3$ thì ta sẽ được một bất đẳng thức mới là $\frac{a}{b}+\frac{b}{c}+\frac{c}{a}\geq a^{2}+b^{2}+c^{2}$
Đến đây mình xin trích dẫn một cách cm bất đẳng thức này của thành viên arqady bên AoPs

 

$a$, $b$ và $c$ là các số dương nên ta đặt

$a+b+c=3u$, $ab+ac+bc=3v^2$, $abc=w^3$ và $u^2=tv^2$.
Do đó, $t\geq1$ và ta cần cm $u^2\sum_{cyc}a^2c\geq3(3u^2-2v^2)w^3$ hoặc $u^2\sum_{cyc}2a^2c\geq6(3u^2-2v^2)w^3$ 
$u^2\sum_{cyc}(a^2b+a^2c)-6(3u^2-2v^2)w^3\geq u^2\sum_{cyc}(a^2b-a^2c)$ 
$u^2(9uv^2-3w^3)-6(3u^2-2v^2)w^3\geq u^2(a-b)(a-c)(b-c)$ 
$3(3u^3v^2-7u^2w^3+4v^2w^3)\geq u^2(a-b)(a-c)(b-c)$.
Ta đi cm $3u^3v^2-7u^2w^3+4v^2w^3\geq0$.
Thật vậy, $(a-b)^2(a-c)^2(b-c)^2\geq0$ gives $27(3u^2v^4-4v^6-4u^3w^3+6uv^2w^3-w^6)\geq0$.
Từ đó, $w^3\leq3uv^2-2u^3+2\sqrt{(u^2-v^2)^3}$.
Vì vậy cần cm: $3u^3v^2\geq(7u^2-4v^2)\left(3uv^2-2u^3+2\sqrt{(u^2-v^2)^3}\right)$ hay
$7u^5-13u^3v^2+6uv^4\geq(7u^2-4v^2)\sqrt{(u^2-v^2)^3}$
$7u^3-6uv^2\geq(7u^2-4v^2)\sqrt{u^2-v^2}$ 
$(21u^4-36u^2v^2+16v^4)v^2\geq0$, hiển nhiên đúng
Do đó ta cần cm
$(3u^3v^2-7u^2w^3+4v^2w^3)^2\geq3u^4(3u^2v^4-4v^6-4u^3w^3+6uv^2w^3-w^6)$, hiển nhiên đúng
$(13u^4-14u^2v^2+4v^4)w^6+3(u^4-5u^2v^2+2v^4)u^3w^3+3u^4v^6\geq0$.
Nếu $u^4-5u^2v^2+2v^4\geq0$ thì bất đẳng thức cuối đúng
Do đó ta phải cm $u^4-5u^2v^2+2v^4<0$ or $1\leq t<\frac{5+\sqrt{17}}{2}$.
Cần cm $3u^6(u^4-5u^2v^2+2v^4)^2-4(13u^4-14u^2v^2+4v^4)u^4v^6\leq0$ hay
$3t(t^2-5t+2)^2-4(13t^2-14t+4)\leq0$,  $(t-1)^2(3t^3-24t^2+36t-16)\leq0$, 
đúng với $1\leq t<\frac{5+\sqrt{17}}{2}$. 
Vậy bất đẳng thức đã được cm


#175
Nguyenhuyen_AG

Nguyenhuyen_AG

    Trung úy

  • Thành viên nổi bật 2016
  • 945 Bài viết

Bài toán :  Cho a,b,c là các số thực dương . CMR : 

                   $\frac{a}{b}+\frac{b}{c}+\frac{c}{a}\geq \frac{9(a^2+b^2+c^2)}{(a+b+c)^2}$

 

P/s : Mình đã nghĩ rất lâu mà không giải được. :angry:

 

Dùng bất đẳng thức này để giải sẽ nhanh hơn

 

\[27(a^2b+b^2c+c^2a+abc) \leqslant 4(a+b+c)^3,\]

với $a,\,b,\,c$ không âm.


Nguyen Van Huyen
Ho Chi Minh City University Of Transport

#176
dangkhuong

dangkhuong

    Sĩ quan

  • Thành viên
  • 312 Bài viết

Cho $a,b,c$ ko âm: $a^3+b^3+c^3+kabc=2$. Tìm hằng số k thực tốt nhất để bất đẳng thức sau đúng :

$a+b+c\leq 2$


:ukliam2:  :ukliam2:  :ukliam2:


#177
Hoang Nhat Tuan

Hoang Nhat Tuan

    Hỏa Long

  • Thành viên
  • 974 Bài viết

 Chuẩn hoá $a+b+c=3$ thì ta sẽ được một bất đẳng thức mới là $\frac{a}{b}+\frac{b}{c}+\frac{c}{a}\geq a^{2}+b^{2}+c^{2}$
Đến đây mình xin trích dẫn một cách cm bất đẳng thức này của thành viên arqady bên AoPs

Em nên dùng BĐT $x^y+y^2z+z^2x+xyz\leq \frac{4(x+y+z)^3}{27}$ thì hay hơn

Áp dụng nó với $x=\frac{a}{b};y=\frac{b}{c};z=\frac{c}{a}$ rồi pqr :luoi:

Spoiler


Bài viết đã được chỉnh sửa nội dung bởi Hoang Nhat Tuan: 28-08-2015 - 17:57

Ngài có thể trói cơ thể tôi, buộc tay tôi, điều khiển hành động của tôi: ngài mạnh nhất, và xã hội cho ngài thêm quyền lực; nhưng với ý chí của tôi, thưa ngài, ngài không thể làm gì được.

#178
HoangVanHaoQBP

HoangVanHaoQBP

    Binh nhì

  • Thành viên
  • 11 Bài viết

Em nên dùng BĐT $x^y+y^2z+z^2x+xyz\leq \frac{4(x+y+z)^3}{27}$ thì hay hơn

Áp dụng nó với $x=\frac{a}{b};y=\frac{b}{c};z=\frac{c}{a}$ rồi pqr :luoi:

Spoiler

quy đồng lên  AM-GM =)))



#179
dogsteven

dogsteven

    Đại úy

  • Thành viên
  • 1567 Bài viết

quy đồng lên  AM-GM =)))

Ta đánh giá mẫu cho $3abc(a+b+c)\leqslant (ab+bc+ca)^3$ rồi chuyển về biến $t=\dfrac{(a+b+c)^2}{ab+bc+ca}$ hay $t=\dfrac{a^2+b^2+c^2}{ab+bc+ca}$ hay đại loại thế.


Quyết tâm off dài dài cày hình, số, tổ, rời rạc.


#180
anhminhnam

anhminhnam

    Trung sĩ

  • Thành viên
  • 155 Bài viết

bạn nào giải bài này hoàn chỉnh mà dễ hiểu với @@ Thắc mắc lâu lắm rồi mà đọc lời giải thì chưa hiểu gì hết.


:like Nếu bạn muốn đến nơi cao nhất, phải học cách bắt đầu từ nơi thấp nhất!  :like 

 





2 người đang xem chủ đề

0 thành viên, 2 khách, 0 thành viên ẩn danh